Find the PDF in terms of another variable

Click For Summary
The discussion focuses on finding the probability density function (PDF) for the variable Y, defined as Y < y = x^2, given the PDF of X. The initial approach incorrectly calculates the cumulative distribution function (CDF) instead of the PDF, leading to confusion about the integration limits. It is clarified that the CDF for Y can be derived from the CDF of X by integrating the PDF of X over the appropriate limits. The correct CDF for Y is F_y(y) = y^2 for 0 ≤ y ≤ 1, and differentiating this gives the PDF f_y(y) = 2y for the same range. This method effectively establishes the relationship between the variables and their respective distributions.
CivilSigma
Messages
227
Reaction score
58

Homework Statement


For

$$f_x(x)=4x^3 ; 0 \leq x \leq 1$$

Find the PDF for $$ Y < y=x^2$$

The Attempt at a Solution


So, we take the domain on x to be:

$$0\leq x \leq \sqrt y$$

and integrate:

$$ \int_0^{\sqrt y} f_x(x) dx = \int_0^{\sqrt y} 4x^3 dx$$

Do we integrate with respect to x or y? I am not sure about this part of the problem.

If we solve the integral above we get:

$$f_x(y)=y^2 ; y \leq x^2 $$

Is this approach correct?

Thank you.
 
Physics news on Phys.org
"So, we take the domain on ##x## to be:"

Wait, what? The domain of ##x## is already defined, it's (0,1]. Why would you change it?

Your integral will is calculating the cumulative distribution of ##x##, the probability that ##x## is less than or equal to ##\sqrt y##. That isn't what was asked for. However ##P(X \leq \sqrt y)## = ##P(X^2 \leq y)## = ##P(Y \leq y)##. So you've calculated the cumulative distribution of ##y##. IOnce you have the CDF for ##y## you can get the ##PDF##.

Now there's a lot wrong with your concluding statement ##f_x(y) = y^2; y \leq x^2##.
- It's not ##f_x## as this is supposed to be the density of ##y##.
- As explained already, you didn't calculate a density, you calculated a CDF.
- Since ##y = x^2## and the domain of ##x## is (0, 1], the domain of ##y## is (0, 1]. The density of ##y## is not related to a particular value of ##x##.

Working with the CDF is a useful thing to do, you're just misinterpreting what every single thing in this calculation is for.

Let's say we want the CDF of ##y##, the probability ##P(Y \leq y)## for any value of ##y##, which as we said could be anything in (0, 1] based on the domain of ##x##. ##Y \leq y \iff X^2 \leq y \iff X \leq \sqrt y##. With a different definition of ##X## that last event might be (##0 \leq X \leq \sqrt y## or ##-\sqrt y \leq X \leq 0##) but X can't be negative here.

OK, so we've established that ##P(Y \leq y) = P(X \leq \sqrt y) = \int_0^{\sqrt y} 4x^3 dx##, and that should address your questions as to whether you're integrating with respect to ##x## or ##y##, and also what you've calculated here and how it relates to what you were asked for.

To summarize: You decide to calculate the cumulative distribution of Y. You want to know how many Y's fall into a given range ##(0,y]##, for any choice of ##y##. You know what X values will give you Y's in the desired range, so you know how to use the density of X to calculate that probability. And that gives you an expression for the CDF of Y.
 
  • Like
Likes CivilSigma
This makes more sense.

So, we first find cumulative probability in terms of the constraint, and then differentiate to get the probability function.

So:

$$F_y(y) = y^2 ; 0 \leq y \leq 1$$
$$f_y (y) = \frac {F_y(y)}{dy} = 2y ; 0 \leq y \leq 1$$
 
Question: A clock's minute hand has length 4 and its hour hand has length 3. What is the distance between the tips at the moment when it is increasing most rapidly?(Putnam Exam Question) Answer: Making assumption that both the hands moves at constant angular velocities, the answer is ## \sqrt{7} .## But don't you think this assumption is somewhat doubtful and wrong?

Similar threads

Replies
7
Views
1K
Replies
11
Views
2K
  • · Replies 105 ·
4
Replies
105
Views
6K
  • · Replies 10 ·
Replies
10
Views
2K
  • · Replies 27 ·
Replies
27
Views
3K
  • · Replies 19 ·
Replies
19
Views
4K
  • · Replies 4 ·
Replies
4
Views
1K
  • · Replies 10 ·
Replies
10
Views
2K
  • · Replies 15 ·
Replies
15
Views
2K
  • · Replies 8 ·
Replies
8
Views
2K